If $a_{n+1}=frac{3+a_n^2}{a_n+1}$ and $a_1=1$, then what is...












3












$begingroup$


Question:




If $a_{n+1}=frac{3+a_n^2}{a_n+1}$ and $a_1=1$, then what is $limlimits_{ntoinfty}left(frac{4}{3}right)^n(3-a_n)$ ?




My approach:



I am able to prove separately that the sequence $a_n$ is convergent and $left(frac{4}{3}right)^n$ is divergent.



But I somehow cannot find out how will their multiplication behave..
enter image description hereenter image description hereenter image description here



But as I cant conclude about the convergence of the entire sequence within the required limit, I cannot find the limit.



Please help



Thank you.










share|cite|improve this question











$endgroup$








  • 1




    $begingroup$
    You cannot write $$ lim_n (4/3)^n (3-a_n) = lim_n (4/3)^n lim_n (3-a_n),$$ because at least one term at the RHS does not exist.
    $endgroup$
    – xbh
    Jan 26 at 9:42






  • 1




    $begingroup$
    Simple example: $(a_n = n)_1^infty$ diverges, $(b_n = 1/n)_1^infty$ converges, but $a_n b_n = 1$ could converge.
    $endgroup$
    – xbh
    Jan 26 at 9:44










  • $begingroup$
    Yes, I get you...Even I doubted this method..Then how can I solve this sum ? Can you help me?
    $endgroup$
    – user515608
    Jan 26 at 10:03






  • 1




    $begingroup$
    "t[h]e sequence has no upper bound as it is decreasing" ?? Every decreasing sequence has a finite upper bound.
    $endgroup$
    – Did
    Jan 26 at 19:28








  • 1




    $begingroup$
    "Am I correct or my thinking direction is wrong?" Again, establishing the convergence is easy but identifying the limit is not. // Say, where did you find this question? I am asking because the gap between the level of the question and the level of your knowledge seems huge...
    $endgroup$
    – Did
    Jan 26 at 19:29


















3












$begingroup$


Question:




If $a_{n+1}=frac{3+a_n^2}{a_n+1}$ and $a_1=1$, then what is $limlimits_{ntoinfty}left(frac{4}{3}right)^n(3-a_n)$ ?




My approach:



I am able to prove separately that the sequence $a_n$ is convergent and $left(frac{4}{3}right)^n$ is divergent.



But I somehow cannot find out how will their multiplication behave..
enter image description hereenter image description hereenter image description here



But as I cant conclude about the convergence of the entire sequence within the required limit, I cannot find the limit.



Please help



Thank you.










share|cite|improve this question











$endgroup$








  • 1




    $begingroup$
    You cannot write $$ lim_n (4/3)^n (3-a_n) = lim_n (4/3)^n lim_n (3-a_n),$$ because at least one term at the RHS does not exist.
    $endgroup$
    – xbh
    Jan 26 at 9:42






  • 1




    $begingroup$
    Simple example: $(a_n = n)_1^infty$ diverges, $(b_n = 1/n)_1^infty$ converges, but $a_n b_n = 1$ could converge.
    $endgroup$
    – xbh
    Jan 26 at 9:44










  • $begingroup$
    Yes, I get you...Even I doubted this method..Then how can I solve this sum ? Can you help me?
    $endgroup$
    – user515608
    Jan 26 at 10:03






  • 1




    $begingroup$
    "t[h]e sequence has no upper bound as it is decreasing" ?? Every decreasing sequence has a finite upper bound.
    $endgroup$
    – Did
    Jan 26 at 19:28








  • 1




    $begingroup$
    "Am I correct or my thinking direction is wrong?" Again, establishing the convergence is easy but identifying the limit is not. // Say, where did you find this question? I am asking because the gap between the level of the question and the level of your knowledge seems huge...
    $endgroup$
    – Did
    Jan 26 at 19:29
















3












3








3


0



$begingroup$


Question:




If $a_{n+1}=frac{3+a_n^2}{a_n+1}$ and $a_1=1$, then what is $limlimits_{ntoinfty}left(frac{4}{3}right)^n(3-a_n)$ ?




My approach:



I am able to prove separately that the sequence $a_n$ is convergent and $left(frac{4}{3}right)^n$ is divergent.



But I somehow cannot find out how will their multiplication behave..
enter image description hereenter image description hereenter image description here



But as I cant conclude about the convergence of the entire sequence within the required limit, I cannot find the limit.



Please help



Thank you.










share|cite|improve this question











$endgroup$




Question:




If $a_{n+1}=frac{3+a_n^2}{a_n+1}$ and $a_1=1$, then what is $limlimits_{ntoinfty}left(frac{4}{3}right)^n(3-a_n)$ ?




My approach:



I am able to prove separately that the sequence $a_n$ is convergent and $left(frac{4}{3}right)^n$ is divergent.



But I somehow cannot find out how will their multiplication behave..
enter image description hereenter image description hereenter image description here



But as I cant conclude about the convergence of the entire sequence within the required limit, I cannot find the limit.



Please help



Thank you.







real-analysis sequences-and-series limits convergence self-learning






share|cite|improve this question















share|cite|improve this question













share|cite|improve this question




share|cite|improve this question








edited Jan 26 at 16:39









rtybase

11.5k31534




11.5k31534










asked Jan 26 at 9:29







user515608















  • 1




    $begingroup$
    You cannot write $$ lim_n (4/3)^n (3-a_n) = lim_n (4/3)^n lim_n (3-a_n),$$ because at least one term at the RHS does not exist.
    $endgroup$
    – xbh
    Jan 26 at 9:42






  • 1




    $begingroup$
    Simple example: $(a_n = n)_1^infty$ diverges, $(b_n = 1/n)_1^infty$ converges, but $a_n b_n = 1$ could converge.
    $endgroup$
    – xbh
    Jan 26 at 9:44










  • $begingroup$
    Yes, I get you...Even I doubted this method..Then how can I solve this sum ? Can you help me?
    $endgroup$
    – user515608
    Jan 26 at 10:03






  • 1




    $begingroup$
    "t[h]e sequence has no upper bound as it is decreasing" ?? Every decreasing sequence has a finite upper bound.
    $endgroup$
    – Did
    Jan 26 at 19:28








  • 1




    $begingroup$
    "Am I correct or my thinking direction is wrong?" Again, establishing the convergence is easy but identifying the limit is not. // Say, where did you find this question? I am asking because the gap between the level of the question and the level of your knowledge seems huge...
    $endgroup$
    – Did
    Jan 26 at 19:29
















  • 1




    $begingroup$
    You cannot write $$ lim_n (4/3)^n (3-a_n) = lim_n (4/3)^n lim_n (3-a_n),$$ because at least one term at the RHS does not exist.
    $endgroup$
    – xbh
    Jan 26 at 9:42






  • 1




    $begingroup$
    Simple example: $(a_n = n)_1^infty$ diverges, $(b_n = 1/n)_1^infty$ converges, but $a_n b_n = 1$ could converge.
    $endgroup$
    – xbh
    Jan 26 at 9:44










  • $begingroup$
    Yes, I get you...Even I doubted this method..Then how can I solve this sum ? Can you help me?
    $endgroup$
    – user515608
    Jan 26 at 10:03






  • 1




    $begingroup$
    "t[h]e sequence has no upper bound as it is decreasing" ?? Every decreasing sequence has a finite upper bound.
    $endgroup$
    – Did
    Jan 26 at 19:28








  • 1




    $begingroup$
    "Am I correct or my thinking direction is wrong?" Again, establishing the convergence is easy but identifying the limit is not. // Say, where did you find this question? I am asking because the gap between the level of the question and the level of your knowledge seems huge...
    $endgroup$
    – Did
    Jan 26 at 19:29










1




1




$begingroup$
You cannot write $$ lim_n (4/3)^n (3-a_n) = lim_n (4/3)^n lim_n (3-a_n),$$ because at least one term at the RHS does not exist.
$endgroup$
– xbh
Jan 26 at 9:42




$begingroup$
You cannot write $$ lim_n (4/3)^n (3-a_n) = lim_n (4/3)^n lim_n (3-a_n),$$ because at least one term at the RHS does not exist.
$endgroup$
– xbh
Jan 26 at 9:42




1




1




$begingroup$
Simple example: $(a_n = n)_1^infty$ diverges, $(b_n = 1/n)_1^infty$ converges, but $a_n b_n = 1$ could converge.
$endgroup$
– xbh
Jan 26 at 9:44




$begingroup$
Simple example: $(a_n = n)_1^infty$ diverges, $(b_n = 1/n)_1^infty$ converges, but $a_n b_n = 1$ could converge.
$endgroup$
– xbh
Jan 26 at 9:44












$begingroup$
Yes, I get you...Even I doubted this method..Then how can I solve this sum ? Can you help me?
$endgroup$
– user515608
Jan 26 at 10:03




$begingroup$
Yes, I get you...Even I doubted this method..Then how can I solve this sum ? Can you help me?
$endgroup$
– user515608
Jan 26 at 10:03




1




1




$begingroup$
"t[h]e sequence has no upper bound as it is decreasing" ?? Every decreasing sequence has a finite upper bound.
$endgroup$
– Did
Jan 26 at 19:28






$begingroup$
"t[h]e sequence has no upper bound as it is decreasing" ?? Every decreasing sequence has a finite upper bound.
$endgroup$
– Did
Jan 26 at 19:28






1




1




$begingroup$
"Am I correct or my thinking direction is wrong?" Again, establishing the convergence is easy but identifying the limit is not. // Say, where did you find this question? I am asking because the gap between the level of the question and the level of your knowledge seems huge...
$endgroup$
– Did
Jan 26 at 19:29






$begingroup$
"Am I correct or my thinking direction is wrong?" Again, establishing the convergence is easy but identifying the limit is not. // Say, where did you find this question? I am asking because the gap between the level of the question and the level of your knowledge seems huge...
$endgroup$
– Did
Jan 26 at 19:29












1 Answer
1






active

oldest

votes


















2












$begingroup$

EDIT: I misunderstood the question. This answer only proves convergence, but doesn't find the limit.



Let's try looking at rations again:



$$
begin{align}
R_n = frac{(frac{4}{3})^{n+1}(3-a_{n+1})}{(frac{4}{3})^{n}(3-a_{n})} &=
frac{4}{3} times frac{3-frac{3+a_n^2}{a_n+1}}{3-a_{n}} =
frac{4}{3} times frac{frac{3a_n + 3 - 3 - a_n^2}{a_n+1}}{3-a_{n}} = \
&= frac{4}{3} times frac{a_n(3-a_n)}{(a_n+1)(3-a_{n})} =
frac{4}{3} times frac{a_n}{a_n+1}
end{align}
$$



We know that $a_n < 3$. Since $frac{x}{x+1}$ is increasing, $frac{a_n}{a_n+1} < frac{3}{4}$. Therefore, $R_n < 1$.



Note that we're not doing ratio test (ratio test is for series, not for sequences), and we don't need to take the limit of $R_n$.



Instead, knowing that $R_1<1$, we can conclude that the sequence is decreasing. It's also positive, so it must converge.






share|cite|improve this answer











$endgroup$













  • $begingroup$
    Thank you..But now that we know, the sequence is convergent, how can I find the given limit, i.e, lim[(4/3)^n](3-a_n) ?
    $endgroup$
    – user515608
    Jan 26 at 10:32








  • 1




    $begingroup$
    Ah, sorry, I don't know how to get that. You should probably unaccept this answer though, as when it's accepted people will this the question resolved.
    $endgroup$
    – Todor Markov
    Jan 26 at 11:01












  • $begingroup$
    Thank you for your answer anyway :)
    $endgroup$
    – user515608
    Jan 26 at 13:17












  • $begingroup$
    I obtained the limit as a convergent infinite series, but they again involve $a_n$'s, so could not the explicit value of it.
    $endgroup$
    – i707107
    Jan 26 at 17:21











Your Answer





StackExchange.ifUsing("editor", function () {
return StackExchange.using("mathjaxEditing", function () {
StackExchange.MarkdownEditor.creationCallbacks.add(function (editor, postfix) {
StackExchange.mathjaxEditing.prepareWmdForMathJax(editor, postfix, [["$", "$"], ["\\(","\\)"]]);
});
});
}, "mathjax-editing");

StackExchange.ready(function() {
var channelOptions = {
tags: "".split(" "),
id: "69"
};
initTagRenderer("".split(" "), "".split(" "), channelOptions);

StackExchange.using("externalEditor", function() {
// Have to fire editor after snippets, if snippets enabled
if (StackExchange.settings.snippets.snippetsEnabled) {
StackExchange.using("snippets", function() {
createEditor();
});
}
else {
createEditor();
}
});

function createEditor() {
StackExchange.prepareEditor({
heartbeatType: 'answer',
autoActivateHeartbeat: false,
convertImagesToLinks: true,
noModals: true,
showLowRepImageUploadWarning: true,
reputationToPostImages: 10,
bindNavPrevention: true,
postfix: "",
imageUploader: {
brandingHtml: "Powered by u003ca class="icon-imgur-white" href="https://imgur.com/"u003eu003c/au003e",
contentPolicyHtml: "User contributions licensed under u003ca href="https://creativecommons.org/licenses/by-sa/3.0/"u003ecc by-sa 3.0 with attribution requiredu003c/au003e u003ca href="https://stackoverflow.com/legal/content-policy"u003e(content policy)u003c/au003e",
allowUrls: true
},
noCode: true, onDemand: true,
discardSelector: ".discard-answer"
,immediatelyShowMarkdownHelp:true
});


}
});














draft saved

draft discarded


















StackExchange.ready(
function () {
StackExchange.openid.initPostLogin('.new-post-login', 'https%3a%2f%2fmath.stackexchange.com%2fquestions%2f3088059%2fif-a-n1-frac3a-n2a-n1-and-a-1-1-then-what-is-lim-limits-n-to%23new-answer', 'question_page');
}
);

Post as a guest















Required, but never shown
























1 Answer
1






active

oldest

votes








1 Answer
1






active

oldest

votes









active

oldest

votes






active

oldest

votes









2












$begingroup$

EDIT: I misunderstood the question. This answer only proves convergence, but doesn't find the limit.



Let's try looking at rations again:



$$
begin{align}
R_n = frac{(frac{4}{3})^{n+1}(3-a_{n+1})}{(frac{4}{3})^{n}(3-a_{n})} &=
frac{4}{3} times frac{3-frac{3+a_n^2}{a_n+1}}{3-a_{n}} =
frac{4}{3} times frac{frac{3a_n + 3 - 3 - a_n^2}{a_n+1}}{3-a_{n}} = \
&= frac{4}{3} times frac{a_n(3-a_n)}{(a_n+1)(3-a_{n})} =
frac{4}{3} times frac{a_n}{a_n+1}
end{align}
$$



We know that $a_n < 3$. Since $frac{x}{x+1}$ is increasing, $frac{a_n}{a_n+1} < frac{3}{4}$. Therefore, $R_n < 1$.



Note that we're not doing ratio test (ratio test is for series, not for sequences), and we don't need to take the limit of $R_n$.



Instead, knowing that $R_1<1$, we can conclude that the sequence is decreasing. It's also positive, so it must converge.






share|cite|improve this answer











$endgroup$













  • $begingroup$
    Thank you..But now that we know, the sequence is convergent, how can I find the given limit, i.e, lim[(4/3)^n](3-a_n) ?
    $endgroup$
    – user515608
    Jan 26 at 10:32








  • 1




    $begingroup$
    Ah, sorry, I don't know how to get that. You should probably unaccept this answer though, as when it's accepted people will this the question resolved.
    $endgroup$
    – Todor Markov
    Jan 26 at 11:01












  • $begingroup$
    Thank you for your answer anyway :)
    $endgroup$
    – user515608
    Jan 26 at 13:17












  • $begingroup$
    I obtained the limit as a convergent infinite series, but they again involve $a_n$'s, so could not the explicit value of it.
    $endgroup$
    – i707107
    Jan 26 at 17:21
















2












$begingroup$

EDIT: I misunderstood the question. This answer only proves convergence, but doesn't find the limit.



Let's try looking at rations again:



$$
begin{align}
R_n = frac{(frac{4}{3})^{n+1}(3-a_{n+1})}{(frac{4}{3})^{n}(3-a_{n})} &=
frac{4}{3} times frac{3-frac{3+a_n^2}{a_n+1}}{3-a_{n}} =
frac{4}{3} times frac{frac{3a_n + 3 - 3 - a_n^2}{a_n+1}}{3-a_{n}} = \
&= frac{4}{3} times frac{a_n(3-a_n)}{(a_n+1)(3-a_{n})} =
frac{4}{3} times frac{a_n}{a_n+1}
end{align}
$$



We know that $a_n < 3$. Since $frac{x}{x+1}$ is increasing, $frac{a_n}{a_n+1} < frac{3}{4}$. Therefore, $R_n < 1$.



Note that we're not doing ratio test (ratio test is for series, not for sequences), and we don't need to take the limit of $R_n$.



Instead, knowing that $R_1<1$, we can conclude that the sequence is decreasing. It's also positive, so it must converge.






share|cite|improve this answer











$endgroup$













  • $begingroup$
    Thank you..But now that we know, the sequence is convergent, how can I find the given limit, i.e, lim[(4/3)^n](3-a_n) ?
    $endgroup$
    – user515608
    Jan 26 at 10:32








  • 1




    $begingroup$
    Ah, sorry, I don't know how to get that. You should probably unaccept this answer though, as when it's accepted people will this the question resolved.
    $endgroup$
    – Todor Markov
    Jan 26 at 11:01












  • $begingroup$
    Thank you for your answer anyway :)
    $endgroup$
    – user515608
    Jan 26 at 13:17












  • $begingroup$
    I obtained the limit as a convergent infinite series, but they again involve $a_n$'s, so could not the explicit value of it.
    $endgroup$
    – i707107
    Jan 26 at 17:21














2












2








2





$begingroup$

EDIT: I misunderstood the question. This answer only proves convergence, but doesn't find the limit.



Let's try looking at rations again:



$$
begin{align}
R_n = frac{(frac{4}{3})^{n+1}(3-a_{n+1})}{(frac{4}{3})^{n}(3-a_{n})} &=
frac{4}{3} times frac{3-frac{3+a_n^2}{a_n+1}}{3-a_{n}} =
frac{4}{3} times frac{frac{3a_n + 3 - 3 - a_n^2}{a_n+1}}{3-a_{n}} = \
&= frac{4}{3} times frac{a_n(3-a_n)}{(a_n+1)(3-a_{n})} =
frac{4}{3} times frac{a_n}{a_n+1}
end{align}
$$



We know that $a_n < 3$. Since $frac{x}{x+1}$ is increasing, $frac{a_n}{a_n+1} < frac{3}{4}$. Therefore, $R_n < 1$.



Note that we're not doing ratio test (ratio test is for series, not for sequences), and we don't need to take the limit of $R_n$.



Instead, knowing that $R_1<1$, we can conclude that the sequence is decreasing. It's also positive, so it must converge.






share|cite|improve this answer











$endgroup$



EDIT: I misunderstood the question. This answer only proves convergence, but doesn't find the limit.



Let's try looking at rations again:



$$
begin{align}
R_n = frac{(frac{4}{3})^{n+1}(3-a_{n+1})}{(frac{4}{3})^{n}(3-a_{n})} &=
frac{4}{3} times frac{3-frac{3+a_n^2}{a_n+1}}{3-a_{n}} =
frac{4}{3} times frac{frac{3a_n + 3 - 3 - a_n^2}{a_n+1}}{3-a_{n}} = \
&= frac{4}{3} times frac{a_n(3-a_n)}{(a_n+1)(3-a_{n})} =
frac{4}{3} times frac{a_n}{a_n+1}
end{align}
$$



We know that $a_n < 3$. Since $frac{x}{x+1}$ is increasing, $frac{a_n}{a_n+1} < frac{3}{4}$. Therefore, $R_n < 1$.



Note that we're not doing ratio test (ratio test is for series, not for sequences), and we don't need to take the limit of $R_n$.



Instead, knowing that $R_1<1$, we can conclude that the sequence is decreasing. It's also positive, so it must converge.







share|cite|improve this answer














share|cite|improve this answer



share|cite|improve this answer








edited Jan 26 at 13:54

























answered Jan 26 at 10:02









Todor MarkovTodor Markov

2,420412




2,420412












  • $begingroup$
    Thank you..But now that we know, the sequence is convergent, how can I find the given limit, i.e, lim[(4/3)^n](3-a_n) ?
    $endgroup$
    – user515608
    Jan 26 at 10:32








  • 1




    $begingroup$
    Ah, sorry, I don't know how to get that. You should probably unaccept this answer though, as when it's accepted people will this the question resolved.
    $endgroup$
    – Todor Markov
    Jan 26 at 11:01












  • $begingroup$
    Thank you for your answer anyway :)
    $endgroup$
    – user515608
    Jan 26 at 13:17












  • $begingroup$
    I obtained the limit as a convergent infinite series, but they again involve $a_n$'s, so could not the explicit value of it.
    $endgroup$
    – i707107
    Jan 26 at 17:21


















  • $begingroup$
    Thank you..But now that we know, the sequence is convergent, how can I find the given limit, i.e, lim[(4/3)^n](3-a_n) ?
    $endgroup$
    – user515608
    Jan 26 at 10:32








  • 1




    $begingroup$
    Ah, sorry, I don't know how to get that. You should probably unaccept this answer though, as when it's accepted people will this the question resolved.
    $endgroup$
    – Todor Markov
    Jan 26 at 11:01












  • $begingroup$
    Thank you for your answer anyway :)
    $endgroup$
    – user515608
    Jan 26 at 13:17












  • $begingroup$
    I obtained the limit as a convergent infinite series, but they again involve $a_n$'s, so could not the explicit value of it.
    $endgroup$
    – i707107
    Jan 26 at 17:21
















$begingroup$
Thank you..But now that we know, the sequence is convergent, how can I find the given limit, i.e, lim[(4/3)^n](3-a_n) ?
$endgroup$
– user515608
Jan 26 at 10:32






$begingroup$
Thank you..But now that we know, the sequence is convergent, how can I find the given limit, i.e, lim[(4/3)^n](3-a_n) ?
$endgroup$
– user515608
Jan 26 at 10:32






1




1




$begingroup$
Ah, sorry, I don't know how to get that. You should probably unaccept this answer though, as when it's accepted people will this the question resolved.
$endgroup$
– Todor Markov
Jan 26 at 11:01






$begingroup$
Ah, sorry, I don't know how to get that. You should probably unaccept this answer though, as when it's accepted people will this the question resolved.
$endgroup$
– Todor Markov
Jan 26 at 11:01














$begingroup$
Thank you for your answer anyway :)
$endgroup$
– user515608
Jan 26 at 13:17






$begingroup$
Thank you for your answer anyway :)
$endgroup$
– user515608
Jan 26 at 13:17














$begingroup$
I obtained the limit as a convergent infinite series, but they again involve $a_n$'s, so could not the explicit value of it.
$endgroup$
– i707107
Jan 26 at 17:21




$begingroup$
I obtained the limit as a convergent infinite series, but they again involve $a_n$'s, so could not the explicit value of it.
$endgroup$
– i707107
Jan 26 at 17:21


















draft saved

draft discarded




















































Thanks for contributing an answer to Mathematics Stack Exchange!


  • Please be sure to answer the question. Provide details and share your research!

But avoid



  • Asking for help, clarification, or responding to other answers.

  • Making statements based on opinion; back them up with references or personal experience.


Use MathJax to format equations. MathJax reference.


To learn more, see our tips on writing great answers.




draft saved


draft discarded














StackExchange.ready(
function () {
StackExchange.openid.initPostLogin('.new-post-login', 'https%3a%2f%2fmath.stackexchange.com%2fquestions%2f3088059%2fif-a-n1-frac3a-n2a-n1-and-a-1-1-then-what-is-lim-limits-n-to%23new-answer', 'question_page');
}
);

Post as a guest















Required, but never shown





















































Required, but never shown














Required, but never shown












Required, but never shown







Required, but never shown

































Required, but never shown














Required, but never shown












Required, but never shown







Required, but never shown







Popular posts from this blog

Can a sorcerer learn a 5th-level spell early by creating spell slots using the Font of Magic feature?

ts Property 'filter' does not exist on type '{}'

mat-slide-toggle shouldn't change it's state when I click cancel in confirmation window